Showing a Group cannot be finitely generated

  • Thread starter Bashyboy
  • Start date
  • Tags
    Group
In summary: If so, then that is not correct. I will be able to find a finite subgroup of... something.In summary, the conjecture is correct, and I have proven it. However, before proceeding any further, I need to make sure that my methods are going to work.
  • #1
Bashyboy
1,421
5
Hello everyone,

I have to demonstrate that the two groups ##(\mathbb{Q'}, \cdot )## and ##(\mathbb{R'}, \cdot )##, where ##\mathbb{Q'} = \mathbb{Q} \setminus \{0\}## and ##\mathbb{R'} = \mathbb{R} \setminus \{0\}##.

While trying to solve this problem, a thought suddenly occurred to me. Here is the conjectured I formulated:

Let ##G## be some group with subgroup ##H##. If ##H## cannot be finitely generated, then neither can ##G##.

Proof: Let us suppose that this is false, that ##H \subset G## is not finitely generated, but ##G## is. Let that finite set which generates all of ##G## be ##S##, so that ##\langle S \rangle = G##. Because ##H## is a subgroup of ##G##, then the set ##H## contains some of ##G##'s elements. Furthermore, because ##G##'s elements are generated by ##S##, and because ##H##'s contains some of the elements of ##G##, then some, if not all, of the elements of ##S## generate ##H##. Seeing as ##S## is a finite set, and any subset of it would also be finite, then ##H## must be finitely generated---which is a contradiction. Therefore, if ##H \subset G## is not finitely generated, then ##G## cannot be.

Supposing that this conjecture is correct, and that I have correctly proven it, I sought out to contrive a simple subgroup of ##\mathbb{Q'}##. For instance, ##\{\frac{1}{2},1, 2\}## has an identity, and each element has an inverse. Then I figured that all I needed to do was enumerate all of the subsets of this set, and show that none of them could generate this subgroup.

After I listed all of them, I realized that I never verified if my subgroup had closure---which it does not...
I decided that, before I tread any further, I ought have some of my work verified, and make sure the methods I choose would avail me any. If not, what methods would you suggest?
 
Physics news on Phys.org
  • #2
Bashyboy said:
Hello everyone,

I have to demonstrate that the two groups ##(\mathbb{Q'}, \cdot )## and ##(\mathbb{R'}, \cdot )##, where ##\mathbb{Q'} = \mathbb{Q} \setminus \{0\}## and ##\mathbb{R'} = \mathbb{R} \setminus \{0\}##.

While trying to solve this problem, a thought suddenly occurred to me. Here is the conjectured I formulated:

Let ##G## be some group with subgroup ##H##. If ##H## cannot be finitely generated, then neither can ##G##.

Proof: Let us suppose that this is false, that ##H \subset G## is not finitely generated, but ##G## is. Let that finite set which generates all of ##G## be ##S##, so that ##\langle S \rangle = G##. Because ##H## is a subgroup of ##G##, then the set ##H## contains some of ##G##'s elements.
This is awkwardly stated. It is NOT just that "H must contain some of G's elements", H must contain nothing but elements of G.

Furthermore, because ##G##'s elements are generated by ##S##, and because ##H##'s contains some of the elements of ##G##, then some, if not all, of the elements of ##S## generate ##H##. Seeing as ##S## is a finite set, and any subset of it would also be finite, then ##H## must be finitely generated---which is a contradiction. Therefore, if ##H \subset G## is not finitely generated, then ##G## cannot be.

Supposing that this conjecture is correct, and that I have correctly proven it, I sought out to contrive a simple subgroup of ##\mathbb{Q'}##. For instance, ##\{\frac{1}{2},1, 2\}## has an identity, and each element has an inverse. Then I figured that all I needed to do was enumerate all of the subsets of this set, and show that none of them could generate this subgroup.

After I listed all of them, I realized that I never verified if my subgroup had closure---which it does not...
I decided that, before I tread any further, I ought have some of my work verified, and make sure the methods I choose would avail me any. If not, what methods would you suggest?
 
  • #3
HallsofIvy said:
This is awkwardly stated. It is NOT just that "H must contain some of G's elements", H must contain nothing but elements of G.

True, but does that fundamentally change my proof?
 
  • #4
I think your conjecture is correct; and those elements of [itex]S[/itex] which generate [itex]H[/itex] are [itex]S\cap H[/itex]. And once you've proven the result for But then your left with finding a subgroup which can't be finitely generated, so it would have to be infinite (or [itex]H[/itex] itself could finitely generate it). And even some countable infinite subgroups like
 
  • #5
dslowik said:
I think your conjecture is correct; and those elements of SS which generate HH are S∩HS\cap H. And once you've proven the result for But then your left with finding a subgroup which can't be finitely generated, so it would have to be infinite (or HH itself could finitely generate it). And even some countable infinite subgroups like

I am not entirely sure of what you are saying. Are you saying that I won't be able to find a finite subgroup of ##\mathbb{Q'}##?
 
  • #6
sorry, i was futzing with some Latex preview bug in trying to edit my reply..

I think your conjecture is correct; and those elements of [itex]S[/itex] which generate [itex]H[/itex] are [itex]S\cap H[/itex]. And once you've proven the result for [itex]\mathbb{Q'}[/itex], you could use that conjecture to show it was true for [itex]\mathbb{R'}[/itex].

But then your left with finding a subgroup which can't be finitely generated, so it would have to be infinite (or [itex]H[/itex] itself could finitely generate it). And even some countable infinite subgroups like [itex]2^{\mathbb{Z}}[/itex] can be finitely generated. beyond that, I'm still thinking..
 
  • #7
The conjecture is wrong. There are finitely generated groups with subgroups which are not finitely generated; take ##G=\langle a,b\rangle## and ##H=\langle\{a^nb^n\}_{n\in\mathbb{N}}\rangle##.

Furthermore, the generators of a group need not be generators for the subgroups, and vice versa; take ##G=(\mathbb{Z},+)=\langle 1\rangle## and ##H=2\mathbb{Z}=\langle2\rangle##.
 
  • #8
Here are two ideas that will lead to proofs, but I can't work through the details now.
1) Any rational, expressed in lowest terms (by dividing common factors top and bottom), num and den can be uniquely factored into primes. So any finite set thereof, has only a finite number of distinct primes in each of num and den. Any rational reachable as some product of integral(+/-) powers has num prime factorization containing only those distinct primes of original set. But there are an infinite number of primes so there is a rational that can't be reached.
So you got the rationals, and your conjecture proves it for the reals. Or,
2) Any product of integral(+/-) powers of a finite collection of N reals, which commute, can be viewed as a finite(N) product of countable(integers) spaces, which is countable, but the reals aren't.

if any questions on working these through let me know.
 
  • #9
gopher_p said:
The conjecture is wrong. There are finitely generated groups with subgroups which are not finitely generated; take ##G=\langle a,b\rangle## and ##H=\langle\{a^nb^n\}_{n\in\mathbb{N}}\rangle##.

Furthermore, the generators of a group need not be generators for the subgroups, and vice versa; take ##G=(\mathbb{Z},+)=\langle 1\rangle## and ##H=2\mathbb{Z}=\langle2\rangle##.

I agree with these two points. Interesting that to specify the structure/constraints of the subgroup in first point takes infinitely many generators, while the containing group has only two generators.
 
  • #10
gopher_p said:
The conjecture is wrong. There are finitely generated groups with subgroups which are not finitely generated

This appears to be different than what I said in my conjecture. I said that, if ##H## is a subgroup of ##G## and it is not finitely generated, then ##G## cannot be finitely generated either. Furthermore, if the conjecture is wrong, then why was I able to arrive the fact with ostensibly sound logic? Where is the error in my proof?
 
  • #11
Bashyboy said:
This appears to be different than what I said in my conjecture. I said that, if ##H## is a subgroup of ##G## and it is not finitely generated, then ##G## cannot be finitely generated either. Furthermore, if the conjecture is wrong, then why was I able to arrive the fact with ostensibly sound logic? Where is the error in my proof?

Your claim is the contrapositive of (and therefore logically equivalent to) the statement "If ##G## is finitely generated, then every subgroup ##H## of ##G## is also finitely generated". I have provided a counterexample to both versions of that claim.

Your error is the part where you say
Bashyboy said:
(S)ome, if not all, of the elements of ##S## generate ##H##.
and interpret that (incorrect) claim as meaning that ##H## is generated by a subset of the generating set for ##G##. I have provided a counterexample to that claim as well.
 
  • #12
Ooh, very lovely. Thank you.
 

Related to Showing a Group cannot be finitely generated

1. What does it mean for a group to be finitely generated?

Being finitely generated means that the group can be generated by a finite set of elements. This means that every element in the group can be expressed as a combination of these finite elements using the group's operation.

2. How do you show that a group cannot be finitely generated?

To show that a group cannot be finitely generated, you must prove that there is no finite set of elements that can generate the entire group. This can be done by showing that the group has an infinite number of elements or that it has infinite subgroups.

3. Can a finite group be finitely generated?

Yes, a finite group can be finitely generated. This means that a finite group can be generated by a finite set of elements, and every element in the group can be expressed as a combination of these finite elements using the group's operation.

4. What is an example of a group that cannot be finitely generated?

The group of real numbers under addition cannot be finitely generated. This is because there is no finite set of real numbers that can generate all real numbers through addition. Similarly, the group of integers under multiplication also cannot be finitely generated.

5. Why is showing that a group cannot be finitely generated important in mathematics?

Showing that a group cannot be finitely generated is important because it proves the existence of infinite structures in mathematics. This allows for a deeper understanding and exploration of infinite groups, which have many applications in various branches of mathematics and other fields such as physics and computer science.

Similar threads

  • Calculus and Beyond Homework Help
Replies
1
Views
575
  • Calculus and Beyond Homework Help
Replies
1
Views
607
  • Calculus and Beyond Homework Help
Replies
1
Views
624
  • Calculus and Beyond Homework Help
Replies
3
Views
861
  • Calculus and Beyond Homework Help
Replies
3
Views
585
  • Calculus and Beyond Homework Help
Replies
4
Views
548
  • Calculus and Beyond Homework Help
Replies
2
Views
416
  • Calculus and Beyond Homework Help
Replies
3
Views
2K
  • Calculus and Beyond Homework Help
Replies
6
Views
889
  • Calculus and Beyond Homework Help
Replies
2
Views
829
Back
Top